It's an article about 2 schools of economics. I don't understand Q15 which answer is A. I chose B.
A. The environment's ability to yield raw material is limited.
B.Natural resources are an external constraint on economics.
I wanted to check my understanding on this question vis. all of yours. I got this question wrong, however, in my blind review, I noted a necessary assumption and wanted to see ...
Can somebody please explain the correct answer choice? I thought all of these answer choice were terrible/unsupported.
http://7sage.com/lsat_explanations/lsat-53-section-4-passage-4-passage/
http://7sage.com/lsat_explanations/lsat-53-section ...
I've done hundreds of LR questions and I was never this lost with a question. Got this right because the rest of the answers weren't compatible with the argument, but I have no idea why (C) is correct. I really would appreciate a detailed explanation for ...
This question went completely over my head and I still don't understand why the correct answer is (C) for all the tea in China. If someone could please explain this question it would be greatly appreciated!
Can some of you awesome people weigh in on statistical reasoning as it's used in the LSAT? For example, PT18 S2 Q4. Specifically, things like probability or likelihood trip me up. Any resources or explanations would be appreciated! Thanks!
I understand why "A" is correct but still cannot arrive at why "E" is incorrect. My thought was that "E" allowed for the possibility that instead of not believing the ...
I know (A) is correct since it points out there may be no relation between jury decision and testimony. Yet, I'm still confused about (B). Is it descriptively wrong since the stimulus only says one fact instead of two facts?
In JYs video explanations, he says that the last 2 aren't applicable to the modern lsat. Given what the exam has been up to in recent years on logic games, do you think that statement still stands? I went -4 in this section and usually do -0/-1 so it threw ...
Hey, guys! I'm having trouble understanding why B is incorrect. My reasoning for selecting B was that if the diseased male grouse were treated with antibiotics their sacs would show that they were infected and this strengthened the hypothesis because the ...
Chose the credited answer, because nothing else came close, but I'm quite bothered with it. Where does the passage say, or even suggest, that the finding about neutrinos might someday be extended to a complete solution of the dark matter problem?
< ...
Can someone please be kind enough to explain to me why the answer is (c)? I was able to eliminate only (a) and (b). I thought (c), (d), and (e) were all valid. I think I am missing something here.. Please help! Thank you so much.
I understand why (A) is correct, as well as why the wrong answers are incorrect. That being said, I'm having trouble identifying the type of flaw/assumption that this argument is making.
Is this a study flaw? Is the author assuming that the ...